polyhedron
LattE background

Applications to Minkowski's Theorem

Theorem 1   Any convex set (or body) in $ {\mathbb{R}}^n$ that has central symmetry and volume greater than $ 2^n$ contains an integer lattice point other than $ \overline{0}$.

If $ A$ is an invertible matrix, then $ Ax$ is a linear map. The lattice $ {\mathbb{Z}}^n$ is mapped into a system of points in $ {\mathbb{R}}^n$ which we call a lattice $ L$, where $ det(A)$ is equal to the determinant of the lattice $ L$.

Theorem 2   Let $ L$ be any lattice $ L$ of determinant $ \Delta\in{\mathbb{R}}^n$. Then any convex set symmetrical about the origin whose volume is greater than $ 2^n\Delta$ contains a point of $ L$ other than $ (0,0)$.

Example 3   Given any real number $ \alpha$ and integer $ t'>0$, there exists integers $ p,q$ such that $ \vert\frac{p}{q}-\alpha\vert \leq \frac{1}{\vert p\vert t'}$.

Take $ M$-set parallelogram bounded by the four lines

$\displaystyle y-\alpha x=k, y-\alpha x =-k, x=-t, x=t
$

where $ t=\vert p\vert t'$.

\includegraphics[width=3 in]{para.eps}

This parallelogram has base $ 2y$, altitude $ 2k$, hence $ Area=2t\cdot2k=4tk$. If we take $ k=\frac{1}{t}$, then $ area=4$. By Minkowski's fundamental theorem, there must be at least one lattice point $ (p,q)$ other than $ (0,0)$. Thus

$\displaystyle \vert p\vert\leq t
$

or,

$\displaystyle \alpha p-k \leq q \leq \alpha p+k.
$

Recall $ k=\frac{1}{t}$, so

$\displaystyle \alpha p-\frac{1}{t} \leq q \leq \alpha p+\frac{1}{t}\Rightarrow \vert q-\alpha p\vert \leq \frac{1}{t}=\frac{1}{\vert p\vert t'}.
$

QED

Example 4   Lagrange: Every positive integer $ n$ can be expressed as a sum of four squares,

$\displaystyle n=x_1^2+x_2^2+x_3^2+x_4^2
$

where $ x_i$ are non-negative integers.

In view of the algebraic identity

  $\displaystyle (x_1^2 + x_2^2 + x_3^2 + x_4^2)(y_1^2 + y_2^2 + y_3^2 + y_4^2)$ $\displaystyle =$ $\displaystyle (x_1 y_1 + x_2 y_2 + x_3 y_3 + x_4 y_4)^2$
      $\displaystyle + (x_1 y_2 - x_2 y_1 + x_3 y_4 - x_4 y_3)^2$
      $\displaystyle + (x_1 y_3 - x_2 y_4 - x_3 y_1 + x_4 y_2)^2$
      $\displaystyle + (x_1 y_4 + x_2 y_3 - x_3 y_2 - x_4 y_1)^2,$

we see that $ m,n$ are sums of four squares, thus $ mn$ is the sum of four squares. Therefore, it is enough to prove the theorem for primes. Let

$\displaystyle A=
\left[
\begin{array}{cccc}
p & 0 & r & s \\
0 & p & s & -r \\
0 & 0 & 1 & 0 \\
0 & 0 & 0 & 1
\end{array}
\right]
$

where $ r,s$ are chosen so that $ r^2 + s^2 + 1 \equiv 0 \,$   mod$ \, p$. Let $ \Lambda=A{\mathbb{Z}}^4$ and suppose that $ x=At$ is a point of $ \Lambda$. If $ x\in\Lambda$, then
  $\displaystyle x_1^2 + x_2^2 + x_3^2 + x_4^2$ $\displaystyle =$ $\displaystyle (pt_1 + rt_3 + st_4)^2 + (pt_2 - st_3 - rt_4)^2 + t_3^2 + t_4^2$
    $\displaystyle \equiv$ $\displaystyle (1+r^2+s^2)(t_3^2+t_4^2) \,$   mod$\displaystyle \, p$
    $\displaystyle \equiv$ $\displaystyle 0 \,$   mod$\displaystyle \, p$   .

So we have $ det(A)=p^2$. Now consider the ball

$\displaystyle \zeta=\{\overline{x} \, \vert \, x_1^2+x_2^2+x_3^2+x_4^2<2p\}.
$

Thus, $ \zeta$ is convex and symmetric about 0. A ball of radius $ r\in{\mathbb{R}}^4$ has volume $ \frac{1}{2}\pi^2 r^4$. If we take $ r=\sqrt{2p}$, we see
  $\displaystyle V(\zeta)$ $\displaystyle =$ $\displaystyle \frac{1}{2}\pi^2 (2p)^2$
    $\displaystyle =$ $\displaystyle 2 \pi^2 p^2$
    $\displaystyle >$ $\displaystyle 2^4 p^2$
    $\displaystyle \Rightarrow$ by Minkowski's Fundamental Theorem $\displaystyle \exists \, \overline{x} \in \Lambda, \: \overline{x} \neq 0, \: \overline{x} \in \zeta$
    $\displaystyle \Rightarrow$ $\displaystyle 0<x_1^2+x_2^2+x_3^2+x_4^2<2p$
    $\displaystyle \equiv$ $\displaystyle 0 \,$   mod$\displaystyle \, p,$

so it has to be $ p$!

QED

Theorem 5   There exists infinitely many positive integers that can be written as a sum of four positive perfect squares, but every integer $ n>169$ is a sum of five positive perfect squares.



• Mathematics • The University of California, Davis •
• One Shields Avenue • Davis, CA 95616 •